LSAT and Law School Admissions Forum

Get expert LSAT preparation and law school admissions advice from PowerScore Test Preparation.

 Nikki Siclunov
PowerScore Staff
  • PowerScore Staff
  • Posts: 1362
  • Joined: Aug 02, 2011
|
#20476
Hi Sherry,

Thanks for your question. The student's response is a classic Straw Man argument: he attacks the teacher's argument by distorting and refashioning it. The teacher never said that journalists need not bother with sources, nor did she suggest that journalists can just invent stories out of thin air. Answer choice (B) gets the heart of this problem: perhaps the teacher would only approve of publishing unattributed statements if such statements were actually made? This answer choice brings up a possibility that, if true, would weaken the student's position, exposing it as a Straw Man argument.

So, you correctly identified the problem (Straw Man), and yet thought that answer choice (D) is attractive. Answer choice (D) neither states nor implies that the student mischaracterized the teacher's position. Just because the student takes the teacher's position to the extreme does not mean that he's judging that position by the most extreme case to which the position applies. Does the teacher's position really apply to inventing stories out of thin air? I don't think so. Twisting a position and taking a position to its logical extreme are very, very different concepts: the first one distorts the conclusion and amounts to a logical fallacy. The second one is not a fallacy - in fact, if taking a position to its logical extreme yields absurd consequences, the position you're attacking may indeed be problematic, in which case your line of attack would be justified.

Hope this clears things up! :-)

Btw, you were asking about Question 17, not 18. I corrected your post title accordingly.
 Sherry001
  • Posts: 81
  • Joined: Aug 18, 2014
|
#20483
Thanks so much
 PeterC123
  • Posts: 26
  • Joined: Dec 27, 2016
|
#31615
Hi,

I am wondering due to the similarities between this question and Prep test 13, section 2, question 26 (blankenship enterprise) question, seeing how both have mistaken reversal flaws and both have answers that are not the typical "mistaken a necessary for a sufficient" answer, can the answer for #17 be "the argument fails to establish that a condition (publication) under which a phenomenon (in this case 3) is said to occur is the only condition under those phenomenon to occur.

The reason I am thinking about this is that in the Blankenship example, they could have multiple sufficient conditions that would force the necessary condition (decrease in profit) to occur, so even if you have the necessary condition, you might have the original sufficient condition (switching suppliers) occurring and other hidden ones, that's the flaw. Therefore, since Q17 follows exactly that line of reasoning, I thought that the right answer for Blankenship would also be right for Q17.

And vice versa? No? For example, you could say for the Blankenship question that the person fails to consider that there are more (hidden) necessary conditions, and they didn't include those and therefore the reasoning is wrong.

Thank you,
 David Boyle
PowerScore Staff
  • PowerScore Staff
  • Posts: 836
  • Joined: Jun 07, 2013
|
#31623
PeterC123 wrote:Hi,

I am wondering due to the similarities between this question and Prep test 13, section 2, question 26 (blankenship enterprise) question, seeing how both have mistaken reversal flaws and both have answers that are not the typical "mistaken a necessary for a sufficient" answer, can the answer for #17 be "the argument fails to establish that a condition (publication) under which a phenomenon (in this case 3) is said to occur is the only condition under those phenomenon to occur.

The reason I am thinking about this is that in the Blankenship example, they could have multiple sufficient conditions that would force the necessary condition (decrease in profit) to occur, so even if you have the necessary condition, you might have the original sufficient condition (switching suppliers) occurring and other hidden ones, that's the flaw. Therefore, since Q17 follows exactly that line of reasoning, I thought that the right answer for Blankenship would also be right for Q17.

And vice versa? No? For example, you could say for the Blankenship question that the person fails to consider that there are more (hidden) necessary conditions, and they didn't include those and therefore the reasoning is wrong.

Thank you,

Hello PeterC123,

There may not be much real resemblance between the Blankenship question and #17, but since you are asking, we shall explore the issue some.
Answer B from #26, "The argument fails to establish that a condition under which a phenomenon is said to occur is the only condition under which that phenomenon occurs", could possibly work, although the wording would be a little strange vis-a-vis #17. Better phrasing for #17 might be, "The argument fails to establish that the necessary conditions (plausible, original, or interesting) for a phenomenon also constitute conditions under which that phenomenon occurs (sufficient conditions)."
You say, "For example, you could say for the Blankenship question that the person fails to consider that there are more (hidden) necessary conditions, and they didn't include those and therefore the reasoning is wrong." Maybe not, since #26 is a pretty standard Mistaken Reversal problem, so that the main thing is that there could be other sufficient conditions that produce the necessary one, besides just the one "has to switch suppliers" sufficient condition.
...Can we refashion answer B for #17, "ignoring the possibility that the teacher regards as a prerequisite for the publication of an unattributed statement that the statement have actually been made", to something fitting #26, e.g., "ignoring the possibility that the speaker regards as a prerequisite for the switching of suppliers that the suppliers actually exist"? Or what would it be, exactly? So answer B for #17 might not travel well to #26. Again, #17 might be more of a "Straw Man" problem than just a standard "Mistaken Reversal" one like #26.

Hope this helps,
David
 akanshalsat
  • Posts: 104
  • Joined: Dec 20, 2017
|
#42712
LRTT 10 #53

Hello! so I correctly categorized this problem as a "Straw Man" one, and though I chose D, reading through your explanations I realize that just because an answer is extreme, doesn't mean its a "distortion". I think my confusion lies in not being able to understand what the answer choices actually mean. Sadly I did not catch the conditionality in the stimulus, and I'm still a bit confused. Super sorry if I'm asking you all to be redundant, but can someone please explain exactly what B is saying and how we were supposed to identify the conditionality in the Teacher's stimulus.
 Adam Tyson
PowerScore Staff
  • PowerScore Staff
  • Posts: 5153
  • Joined: Apr 14, 2011
|
#42718
We're happy to try providing another way of looking at the question, akanshalsat! There are always multiple ways to get to the right answer, and to understanding it.

One key to the conditional nature of the Teacher's position is through the use of conditional indicators. In this case, the first one is a bit subtle - "journalists who" is a variant of "people who", and is what I like to refer to as a classic "category/characteristic" scenario. Any time an author describes a category of things and then assigns that entire category a characteristic, you have a conditional relationship - if you are in the category, then you have the characteristic. Here, the category is "journalists who conceal their sources" and the characteristic is "stake their reps on anecdotes" (I'm abbreviating a bit). Then later in the stimulus we have a clear "only if" statement - acceptance for publication is sufficient for being high in plausibility or originality or interest to a given audience.

The student then confuses the conditional nature of that latter claim, and acts as of the Teacher has said that being high in plausibility or originality or interest to a given audience would be sufficient for accepting that statement for publication. This is a Mistaken Reversal of what the teacher actually said, and that is in part what answer B is describing.

How do we know that answer B is dealing with conditionality? Because it uses a variant on another classic indicator, "required". That variant is "prerequisite". Saying that one thing is a prerequisite for another is the same as saying that thing is necessary. The student has acted as if a plausible, original, or interesting statement is sufficient for publication, and that there is no other necessary condition, like having actually been said. If being actually said is necessary, then the student's response falls apart!

I hope that helped!
User avatar
 pbui5300
  • Posts: 7
  • Joined: Oct 16, 2021
|
#91770
Hello,

I was practicing this question in the "Strengthen Questions: Vol 1" drill test of your LSAT Testing and Analytics Package. The correct answer shown in there is A but the answer here in this discussion is B. I googled this question and some sources said A and some said B. Can you clarify the correct answer? Thank you in advance!
User avatar
 Stephanie Oswalt
PowerScore Staff
  • PowerScore Staff
  • Posts: 811
  • Joined: Jan 11, 2016
|
#91803
pbui5300 wrote: Tue Nov 02, 2021 2:12 pm Hello,

I was practicing this question in the "Strengthen Questions: Vol 1" drill test of your LSAT Testing and Analytics Package. The correct answer shown in there is A but the answer here in this discussion is B. I googled this question and some sources said A and some said B. Can you clarify the correct answer? Thank you in advance!
Hi pbui!

There are actually two questions on PT11 with the same stimuli! :-D

The answer to this one, #17, with the question stem of, "The student's response contains... " is B.

The answer to #18, with the question stem of, "Which one of the following, if true..." is A. I think you're looking at #18. You can find the forum thread for that one here: https://forum.powerscore.com/viewtopic ... 75&t=14430.

Hope this helps clear up the confusion! :)
User avatar
 pbui5300
  • Posts: 7
  • Joined: Oct 16, 2021
|
#91846
Thanks so much Stephanie! I was not aware there were two questions with the same stimuli.
 Mastering_LSAT
  • Posts: 35
  • Joined: Jul 30, 2020
|
#96050
Hello PowerScore,

I have a quick question.

Why do we have to assume that when the teacher specified requirements for publication through "only if" requirement modifier (plausibility, or originality, or interest), we should treat this case as open to the possibilities that the teacher may have in mind other requirements that s/he did not explicitly specified vs the only requirements for publication? Should we always treat "only if" requirements in other arguments as open to the possibility that other requirements that the author did not explicitly specified in the argument may still apply?

Sometimes on LSAT we have to assume that when the author mentions something it is the only thing the author believes in and anything else (i.e., non-mentioned) the author does not believe in. A basic causation assumption comes to mind as an example.

Thank you!

Get the most out of your LSAT Prep Plus subscription.

Analyze and track your performance with our Testing and Analytics Package.